Sunteți pe pagina 1din 26

With a conflict of interest analysis do a Confidentialityanalysis

PERSONAL AND PROFESSIONAL RESPONSIBILITY

DISCIPLINARY PROCESS: Professional discipline means punishment imposed on a lawyer for breaking a rule of professional ethics - Complaint: o Disciplinary proceedings against a lawyer begin when a compliant is made to the state disciplinary authority (usually the bar). Most claims are filed by aggrieved clients, but may also be brought by anyone with knowledge of the misconduct. Filing a complaint against a lawyer is considered privileged, and thus cannot be the basis of action by the lawyer against the complaint - Screening: o If the complaint is without merit, it might e dismissed by the grievance committee w/o ever involving a lawyer. If the complaint appears to have merit, the lawyer will be asked to respond to the charges. After further investigation, the committee will either dismiss the complaint or schedule a hearing. If the committee dismisses the complaint, the complainant does not have any right to appeal, the decision is final - Hearing: o If there is a hearing on the complaint, the accused lawyer is entitled to procedural due process, which means that she has the right to counsel, to proper notice, to be heard and introduce evidence, and to cross-examine adverse witnesses. The hearing must also be limited to the charges made in the complaint - Burden of Proof: o The burden of proof is on the party prosecuting the charge, and most states requires proof of the charge beyond a preponderance of the evidence but less than beyond a reasonable doubt. Only evidence admissible under the rules of evidence are allowed to be introduced, hearsay is excluded - Decision and Review: o After the hearing the grievance committee will either dismiss the charges or recommend sanctions. If sanctions are recommended or disciplinary action is actually taken the lawyer may appeal the decision to the states highest court. The burden then switches to the lawyer to prove that the committees actions or recommendations is not supported by the record or is otherwise unlawful - Sanctions: the most common sanctions imposed on a lawyer found to have committed professional misconduct are: o Private or Public Reprimand or Censure: this is simply acknowledgement of misconduct that goes on the lawyers record with the disciplinary authorities o Suspension: the lawyers license to practice for a definite period of time, at the end of which the right to practice is automatically reinstated o Disbarment: this is permanent revocation of the lawyers license to practice. A disbarred lawyer may, however, apply for readmission upon proof of rehabilitation o Other Sanctions: these may include probation, restitution, costs of the disciplinary proceedings, and limitations on the lawyers practice. The sanction that is imposed depends upon the severity of the misconduct and the presence of absence of mitigating or aggravating circumstances CHOICE OF LAW IN DISCIPLINARY PROCEEDINGS - If the conduct in question occurredin connection with a proceeding that is pending before a tribunal, the ethics rules of the jurisdiction in which the tribunal sits will be used, unless the tribunals rules provide otherwise o ABA Rule 8.5(b)(1)  For any other conduct, the rules of the jurisdiction in which the conduct occurred will apply, but if the predominant effect of the conduct is in some other jurisdiction, that jurisdictions rules will apply. (b)(2) states that a lawyer will not be subject to discipline if her conduct is proper in the jurisdiction in which she reasonably believes the predominant effect of her conduct will occur - Effect of Sanctions in Other Jurisdictions o A suspicion or disbarment in one jurisdiction does not automatically affect a lawyers ability to practice in another jurisdiction. Professional discipline imposed in on state does not necessarily mean that it is binging in another. Most states accept the discipline as proof of misconduct but the sanctions may not be UNAUTHORIZED PRACTICE AND MULTI-JURISDICTIONL PRACTICE - Unauthorized Practice By A Lawyer o A lawyer who is admitted to practice law in one jurisdiction is not, without more authorized to practice in any other jurisdiction. A lawyer is subject to discipline for practicing in a jurisdiction where she is not admitted to practice o 5.5(b): A lawyer not licensed in the jurisdiction at hand shall not:  Set up an office or other continuous presence in the jurisdiction to practice law, OR  State or imply that the lawyer is authorized to practice in that jurisdiction

Except as allowed by that jurisdictions laws or ethics rules, the unadmitted lawyer must not, represent that she is admitted to practice in that jurisdiction, or establish an office or other systematic or continuous presence for the practice of law in that jurisdiction y Prevents an attorney from helping a non-attorney in the unauthorized practice of law. Permissible Types of Temporary Multi-Jurisdictional Practice: the nature of modern law and commerce requires many lawyers to practice across state lines o ABA Model Rule 5.5(c): Recognizes this fact and provides that if a lawyer is admitted to practice in one state, and is not disbarred or suspended from practice in any state, then she may provide legal services in the second state on a TEMPORARY bases in four situations  5.5(c)(1): Associated with a lawyer y A lawyer may practice in a temporary basis in a state in which she is not admitted if she associates with lawyer who is admitted to practice in that jurisdiction and who ACTIVELY participates in the matter  5.5(c)(2): Special Permission to Practice in Local Tribunal y Pro Hoc Vice (admission for purposes of this matter only): An out of state lawyer may request special permission from a local court administrative agency, or other tribunal to handle a matter in that tribunal. o An out of state lawyer who REASONABLY expects to be admitted pro hoc vice may engage in preliminary activities in the state, such as meeting with clients, reviewing documents, and interviewing witnesses.  5.5(c)(3): Mediation or Arbitration Arising Out of Home State Practice y A lawyer may mediate, arbitrate, or engage in another form of alternative dispute resolution in a state in which she is not admitted to practice if her services arise out of, or are reasonably related to, her practice in the state in which she is admitted  Other Temporary Practice y 5.5(c)(4) is a catch all category that permits a lawyer to temporarily practice out of state if the lawyers out of state practice is reasonably related to the lawyers home state practice Permissible Types of Permanent Multi-Jurisdictional Practices: o Lawyers Employed By Their Own Client  Some lawyer are salaried employees of their only client, such as in house corporate lawyers and lawyers employed by the government. They may set up a permanent office to render legal services to their employer in a state which they are not admitted to practice, but if they want to litigate a matter in that state, they must seek pro hoc vice admission o Legal Services Authorized by Federal or Local Law  5.5(d)(2): In rare instances, federal or local law authorizes a lawyer to practice a restricted branch of law in a state in which he is not otherwise admitted to practice Consequences of Multi-Jurisdictional Practice o A lawyer who is admitted to practice in one state only, but who practices in another state pursuant to b or c headlines above, will be subject to disciplinary rules of both states.  An in house or government lawyer who practices under 8.5(c)(1) may be subject to the second states client security assessments and continuing legal education requirements Unauthorized Practice by Non-Lawyers o A person not admitted to practice as a lawyer must not engage in the unauthorized practice of law, and a lawyer must not assist such a person to do so o General considerations in defining Practice of Law  Whether the activity involves legal knowledge and skill beyond that which the average layperson possesses, whether the activity constitute advice or services concerning binding legal rights or remedies; and whether the activity is one traditionally performed by lawyers  Activities Constituting Law Practice y Courts have found the following constitutes the practice of law when done on behalf of another and include appearing in judicial proceedings, engaging in settlements and negotiations, drafting documents such as wills, trusts, estate planning and courts have also held that non-lawyer clinics on how to obtain a low cost divorce constitute the unauthorized practice of law.  Activities Not Constituting the Practice of Law y Non-lawyers may not draft legal documents, but they can act as scriveners, filling in the blanks on standard forms. Thus, real estate brokers, title insurance companies, and escrow companies are usually permitted to fill in the blanks on standard documents related to the sale of real property y

Tax Advice y Giving advice on Tax Law would probably constitute the unauthorized practice of law, but an accountant or other layperson may prepare tax returns and answer questions incidental to the preparation of the returns - Consequences of the Unauthorized Practice o A non-lawyer who engages in the unauthorized practice of law is subject to several sanctions, including injunction, contempt, and criminal conviction. A lawyer who assists in such an endeavor is subject to professional discipline - Delegating Work to Non-Lawyer Assistants o A lawyer can delegate tasks to a paralegal, law clerk, student intern, or other such person. But the lawyer must supervise the delegated work carefully and must be ULTIMATELY RESPONSIBLE for the results - Training Non-Lawyers for Law Related Work (5.5 Comment 3) o A lawyer may advise and instruct non-lawyers whose employment requires knowledge of the law, such as claim adjusters, bank trust officers, social workers, accountants, and government employees. o A lawyer may also assist an independent non-attorney paraprofessional who are allowed by a jurisdictions law to provide law-related services - Helping Persons Appear Pro Se o A lawyer may advise persons who wish to appear on their own behalf in a legal matter o Must be careful b/c they frequently dont follow instructions and then you are the one who is on the hook - Assisting a Suspended or Disbarred Lawyer o A lawyer violates ABA Model Rule 5.5(a) if he assists a lawyer whose license has been suspended or revoked in practicing law. It is proper to hire a suspended or disbarred lawyer to do the work that a layperson is permitted to do, but the suspended or disbarred lawyer must not be permitted to do any work that constitutes the practice of law THE LAWYER-CLIENT RELATIONSHIP GENERAL POINTS - Nature of the Relationship o The relationship b/w client and lawyer is contractual. The lawyer operates as both the clients fiduciary and agent with the duties of limitations of those designations. - Creating the Lawyer-client Relationship o In the US lawyers are generally free to refuse service to ay person for any reason. A lawyer-client relationship arises when:  A person manifests an intent that the lawyer provide legal services and the lawyer agrees  A person manifests an intent to have the lawyer represent him, the lawyer fails to make clear that he does not want to undertake the representation, and the lawyer knows or should know that the prospective client in REASONABLY RELYING on the lawyer to provide the services, or  A tribunal APPOINTS a lawyer to represent a client - Implied Assent and Reasonable Reliance o The lawyers assent to represent a party is implied when he fails to clearly decline representation and the prospective client reasonably relies on the representation. The reasonableness of the reliance is a question of fact ATTORNEY MALPRACTICE - A MR violation, in itself, does not create a malpractice claim, a presumption of breach of duty in a malpractice case, warrant other nondisciplinary remedy like removal of an attorney from a pending case. o The MR are to provide guidance to lawyers, create a structure to regulate lawyer conduct and are not a basis for civil liability, but a MR violation could be evidence of a breach of applicable standard of care thus leasing to a malpractice suit for breach of fiduciary duty. - In a legal malpractice action, four elements must be shown: o That an attorney-client relationship existed o That acted negligently or in breach of contract o That such acts were the proximate cause of the s damages o That but for s conduct, the would have been successful in the prosecution of their claim - An ineffective assistance of counsel claim in a criminal case must satisfy 2 components: o The must how that counsels performance was deficient. This requires showing that counsel made errors so serious that counsel was not functioning as the counsel o The must show that the deficient performance prejudiced the defense. This requires showing that counsels errors were so serious as to deprive the of a fair trial, whose result is reliable - Suits by Non-Clients: o Some courts have held that the opposing party in litigation cannot sue for malpractice, b/c there is no actionable duty to opposing parties o When the lawyer drafts an opinion knowing that it will be used by non-clients, those non-clients may be able to sue for malpractice 3

Competence, Diligence, Communication, Independent Judgment, Candid advice - Layers have duties even bore or whether formal rep starts - Legal knowledge, skill, thoroughness and preparation needed in the matter - Competence does not require experience. The ability to study and gain the expertise is sufficient. - Act with reasonable diligence and promptness. - 1.4: Inform, consult, explain Communication is always the lawyers responsibility - 1.2 and 2.1: Independent judgment & candid advice; other considerations allowed

MODEL RULES OF PROFESSIONAL CONDUCT


CLIENT-LAWYER RELATIONSHIP Rule 1.0: Terminology a) "Belief" or "believes" denotes that the person involved actually supposed the fact in question to be true. A person's belief may be inferred from circumstances. b) "Confirmed in writing," when used in reference to the informed consent of a person, denotes informed consent that is given in writing by the person or a writing that a lawyer promptly transmits to the person confirming an oral informed consent. See paragraph (e) for the definition of "informed consent." If it is not feasible to obtain or transmit the writing at the time the person gives informed consent, then the lawyer must obtain or transmit it within a reasonable time thereafter. c) "Firm" or "law firm" denotes a lawyer or lawyers in a law partnership, professional corporation, sole proprietorship or other association authorized to practice law; or lawyers employed in a legal services organization or the legal department of a corporation or other organization. d) "Fraud" or "fraudulent" denotes conduct that is fraudulent under the substantive or procedural law of the applicable jurisdiction and has a purpose to deceive. e) "Informed consent" denotes the agreement by a person to a proposed course of conduct after the lawyer has communicated adequate information and explanation about the material risks of and reasonably available alternatives to the proposed course of conduct. f) "Knowingly," "known," or "knows" denotes actual knowledge of the fact in question. A person's knowledge may be inferred from circumstances. g) "Partner" denotes a member of a partnership, a shareholder in a law firm organized as a professional corporation, or a member of an association authorized to practice law. h) "Reasonable" or "reasonably" when used in relation to conduct by a lawyer denotes the conduct of a reasonably prudent and competent lawyer. i) "Reasonable belief" or "reasonably believes" when used in reference to a lawyer denotes that the lawyer believes the matter in question and that the circumstances are such that the belief is reasonable. j) "Reasonably should know" when used in reference to a lawyer denotes that a lawyer of reasonable prudence and competence would ascertain the matter in question. k) "Screened" denotes the isolation of a lawyer from any participation in a matter through the timely imposition of procedures within a firm that are reasonably adequate under the circumstances to protect information that the isolated lawyer is obligated to protect under these Rules or other law. l) "Substantial" when used in reference to degree or extent denotes a material matter of clear and weighty importance. m) "Tribunal" denotes a court, an arbitrator in a binding arbitration proceeding or a legislative body, administrative agency or other body acting in an adjudicative capacity. A legislative body, administrative agency or other body acts in an adjudicative capacity when a neutral official, after the presentation of evidence or legal argument by a party or parties, will render a binding legal judgment directly affecting a party's interests in a particular matter. n) "Writing" or "written" denotes a tangible or electronic record of a communication or representation, including handwriting, typewriting, printing, photostating, photography, audio or videorecording and e-mail. A "signed" writing includes an electronic sound, symbol or process attached to or logically associated with a writing and executed or adopted by a person with the intent to sign the writing. Rule 1.1 Competence - A lawyer shall provide competent representation to a client. Competent representation requires the legal knowledge, skill, thoroughness and preparation reasonably necessary for the representation NOTES - To determine if a lawyer employs the requisite knowledge and skills for a particular matter will depend upon the complexity of the matter, the lawyers general experience, how much time the attorney would have to prepare and whether they could consult with others regarding the matter. The required proficiency is normally that of a general practitioner

A lawyer may provide adequate representation in a wholly novel field through necessary study. Competent representation can also be provided through the association of a lawyer of established competence in the field in question In an emergency a lawyer may give advice or assistance in a matter in which the lawyer does not have the skill ordinarily required where referral to or consultation or association with another lawyer would be impractical. Even in an emergency it should be limited to that reasonably necessary in the circumstances Competence does not require experience. The ability to study and gain the expertise is sufficient.

Rule 1.2 Scope Of Representation And Allocation Of Authority Between Client And Lawyer a. Subject to paragraphs (c) and (d), a lawyer shall abide by a client's decisions concerning the objectives of representation and, as required by Rule 1.4, shall consult with the client as to the means by which they are to be pursued. A lawyer may take such action on behalf of the client as is impliedly authorized to carry out the representation. A lawyer shall abide by a client's decision whether to settle a matter. In a criminal case, the lawyer shall abide by the client's decision, after consultation with the lawyer, as to a plea to be entered, whether to waive jury trial and whether the client will testify. b. A lawyer's representation of a client, including representation by appointment, does not constitute an endorsement of the client's political, economic, social or moral views or activities. c. A lawyer may limit the scope of the representation if the limitation is reasonable under the circumstances and the client gives informed consent. d. A lawyer shall not counsel a client to engage, or assist a client, in conduct that the lawyer knows is criminal or fraudulent, but a lawyer may discuss the legal consequences of any proposed course of conduct with a client and may counsel or assist a client to make a good faith effort to determine the validity, scope, meaning or application of the law. NOTES - This rule essentially says that the client has the ultimate authority to determine the purpose of the legal representation (settle, plea, who they want to sue). Discuss 1.2 scope when the attorney and client initially are deciding what will be done in the case and refer to rule 1.4 regarding the lawyers duty to communicate with the client regarding these decisions. - If there is any disagreement from this initial scope question and either the lawyer wants to withdraw (1.16(b)(4) Permissive) or if the clients want to fire the attorney (1.16(a)(3) mandatory withdrawal) - If there is a client that looks to have diminished capacity reference 1.2, 1.14 regarding diminished capacity. Need to say that a normal attorney-client relationship (still discuss 1.2 and communicate as well as you can under 1.4 unless it becomes such that a guardian should be appointed to impaired) would exist as per 1.6 - Comments 9 & 10 o Say that the lawyer can give his honest view about the actual consequences that appear likely to result from the clients actions o But note that if a lawyer discovers ongoing criminal or fraudulent activity by the client, the lawyer cant continue to help a client engaged in such conduct Rule 1.3 Diligence - A lawyer shall act with reasonable diligence and promptness in representing a client. NOTES - The clients interest should be placed above personal conveniences of the lawyer. Need to act as an advocate on the clients behalf, but this does not imply that you do everything that the client asks such as not granting opposing counsel time or courtesies. - Remember to discuss 1.2 here in that the attorney determines the means by which the matter should be pursued, not the client. - Unless for some reason the representation is terminated under 1.16 a lawyer should carry through to conclusion all matters undertaken for a client. If the lawyers employment was limited to a specific matter, such as a bail hearing, the relationship terminates when the matter is resolved, but if it is a client which you frequently do continued work for, the client sometimes may assume the lawyer will continue to serve on a continuing basis unless the lawyer gives notice of withdrawal (thats why it needs to be in writing) - For there to be true diligence, a sole practitioner would need to have a plan in place in case of his death or disability of the lawyer so that the duty of diligence can be carried through by a competent attorney that may be designated Rule 1.4 Communication a. A lawyer shall: 1. promptly inform the client of any decision or circumstance with respect to which the client's informed consent, as defined in Rule 1.0(e), is required by these Rules; 2. reasonably consult with the client about the means by which the client's objectives are to be accomplished; 3. keep the client reasonably informed about the status of the matter; 4. promptly comply with reasonable requests for information; and

5. consult with the client about any relevant limitation on the lawyer's conduct when the lawyer knows that the client expects assistance not permitted by the Rules of Professional Conduct or other law. b. A lawyer shall explain a matter to the extent reasonably necessary to permit the client to make informed decisions regarding the representation. Rule 1.5 Fees a. A lawyer shall not make an agreement for, charge, or collect an unreasonable fee or an unreasonable amount for expenses. The factors to be considered in determining the reasonableness of a fee include the following: 1. the time and labor required, the novelty and difficulty of the questions involved, and the skill requisite to perform the legal service properly; 2. the likelihood, if apparent to the client, that the acceptance of the particular employment will preclude other employment by the lawyer; 3. the fee customarily charged in the locality for similar legal services; 4. the amount involved and the results obtained; 5. the time limitations imposed by the client or by the circumstances; 6. the nature and length of the professional relationship with the client; 7. the experience, reputation, and ability of the lawyer or lawyers performing the services; and 8. whether the fee is fixed or contingent. b. Scope of Representation and Basis or Rate of Fee: The scope of the representation and the basis or rate of the fee and expenses for which the client will be responsible shall be communicated to the client, preferably in writing, before or within a reasonable time after commencing the representation, except when the lawyer will charge a regularly represented client on the same basis or rate. Any changes in the basis or rate of the fee or expenses shall also be communicated to the client. c. Contingency Fees: A fee may be contingent on the outcome of the matter for which the service is rendered, except in a matter in which a contingent fee is prohibited by paragraph (d) or other law. o A contingent fee agreement shall be in a writing signed by the client and shall state the method by which the fee is to be determined, including the percentage or percentages that shall accrue to the lawyer in the event of settlement, trial or appeal; litigation and other expenses to be deducted from the recovery; and whether such expenses are to be deducted before or after the contingent fee is calculated. o The agreement must clearly notify the client of any expenses for which the client will be liable whether or not the client is the prevailing party. o Upon conclusion of a contingent fee matter, the lawyer shall provide the client with a written statement stating the outcome of the matter and, if there is a recovery, showing the remittance to the client and the method of its determination. d. Prohibition of Contingency Fees: A lawyer shall not enter into an arrangement for, charge, or collect: 1. any fee in a domestic relations matter, the payment or amount of which is contingent upon the securing of a divorce or upon the amount of alimony or support, or property settlement in lieu thereof; or 2. a contingent fee for representing a defendant in a criminal case. e. Division of Fees: A division of a fee between lawyers who are not in the same firm may be made only if: 1. the division is in proportion to the services performed by each lawyer or each lawyer assumes joint responsibility for the representation; 2. the client agrees to the arrangement, including the share each lawyer will receive, and the agreement is confirmed in writing; and 3. the total fee is reasonable. NOTES - True Retainers o The client pays the lawyer to remain ready to provide services, not for the actual provision of services. Also called engagement retainers. o The lawyer does not have to perform services to earn the retainer; just be available to provide legal services. This type of retainer must be deposited into the lawyers general account because it belongs to the lawyer upon receipt o A true retainer can be unreasonable o Once the lawyer performs a service, fees can be billed in addition to the retainer - Advance Fee Payments o If the retainer is an advance against future fees, it is actually called advance fee payment o This advance fee payment must be deposited into a trust account, b/c it is the property of the client until the lawyer earns an interest in it. o Under MR 1.16, the attorney must refund any advance payment of fee that has not been earned upon termination of representation

Clients can also advance money to pay costs or expenses of law suits. These too must be deposited into the trust account b/c they are the property of the client.

Rule 1.6 Confidentiality Of Information a. A lawyer shall not reveal information relating to the representation of a client unless the client gives informed consent, the disclosure is impliedly authorized in order to carry out the representation or the disclosure is permitted by paragraph (b). b. A lawyer may reveal information relating to the representation of a client to the extent the lawyer reasonably believes necessary: 1. to prevent reasonably certain death or substantial bodily harm; 2. to prevent the client from committing a crime or fraud that is reasonably certain to result in substantial injury to the financial interests or property of another and in furtherance of which the client has used or is using the lawyer's services;  the crime or fraud is going to happen and by disclosing the attorney can possibly prevent the crime  the client uses the lawyers services  Related rules if this happens: 1.2(d), 1.16, 1.13(c) 3. to prevent, mitigate or rectify substantial injury to the financial interests or property of another that is reasonably certain to result or has resulted from the client's commission of a crime or fraud in furtherance of which the client has used the lawyer's services;  This is where a lawyer services have beenused to commit a crime or fraud and the lawyer can disclose to prevent  If you do not disclose, the 3rd party can sue the lawyer  Allows the attorney to tell the client you can tell a third party and client may rethink 4. to secure legal advice about the lawyer's compliance with these Rules;  This is where you talk to a 3rd party attorney about a possible MR violation  Lawyer may ask another lawyer about possible model rules violation  You can talk to the 3rd party attorney only about the rules violation and not about the case. The 3rd party is obligated to follow 1.6 so the 3rd party cannot disclose under 1.6 5. to establish a claim or defense on behalf of the lawyer in a controversy between the lawyer and the client, to establish a defense to a criminal charge or civil claim against the lawyer based upon conduct in which the client was involved, or to respond to allegations in any proceeding concerning the lawyer's representation of the client; or 6. To comply with other law or a court order. NOTES - Privileges: Protects the client from compelled disclosure by the lawyer. The privilege analysis is applicable when the twin powers of subpoena and contempt are present o ELEMENTS (if 1 of these are missing, then it is only 1.6)  Communication (can b in any form. Usually oral or written)  Between privileged persons (Lawyer, client and lawyers agents)  For the purposes of securing or delivering legal advise (this even works with what the attorney tells the client)  Made with the expectation of confidentiality - The lawyer must be acting as a lawyer - The privilege belongs to the client; not the lawyer - Communications and observations made as the result of privileged communications are protected, buy physical evidence is not. o The communication must be intended to be in confidence n/a to communications that the clients intends to broadcast or pass on - Who is a privileged person in the corporate setting? o Assuming all other elements of a privilege claim are present, communications with lawyers are privileged when:  Made by employees within the scope of their employment;  At the direction of management;  For the purpose of securing legal advice or services (i.e. evaluating the legality of conduct, determining its consequences, and determining appropriate responses to it.) o You cannot apply 1.13 to a past crime o A lawyer is not able to use 1.13 door if the corporation is trying to fix the problem, you cannot reveal - The privilege protects against compelled disclosures unless: o Client waives or consents 7

This happens when a client talks about the case, they may waive privilege and it can only be 1.6. It may also be used against them o Lawyers services are sought or used in furtherance of a crime or fraud o Joint clients  When you represent 2 clients in the same matter and one will be offered a deal to testify against the other co- , once the co- becomes adverse, the privilege is gone o Testimony relates to documents attested to by lawyer  Ex: will, affidavit, deed  This is where an attorney witnesses a clients signature and in order to validate it the attorney must disclose what they saw o Breach of duty by client or lawyer (self-defense; case against client) o Claimant through the same dead client (Will/Probate)  This is where an attorney drafts a will and knows the intent of the decedent. In order to divide the property through probate, the lawyer will have to testify about what his client said and intended to do The Privilege survives the clients death o Documents and tangible things otherwise discoverable and prepared in anticipation of litigation or for trial by or for another party or by or for that other partys representation are only discoverable if the party seeking discovery o Has substantial need of the materials for trial preparation the party is unable without undue hardship to obtain the substantial equivalent of the material by other means o Attorneys person notes are not discoverable Policy behind confidentiality and privilege o Self-incrimination o Preserve effective representation 1.7 is used to determine if a
Formula: if you have 1.7(a)1 or (a)2 then 1.7b (1-4) must be satisfied in order to represent all of the clients former client was still a client.

Rule 1.7 Conflict Of Interest: Current Clients a. Except as provided in paragraph (b), a lawyer shall not represent a client if the representation involves a concurrent conflict of interest. A concurrent conflict of interest exists if: 1. the representation of one client will be directly adverse to another client; or  Direct Adversity y A lawyer may not act as an advocate in one matter against a person the lawyer represented in some other matter, even when the matters are wholly unrelated If you dont y Look at if the prospective clients interests will hurt your current clients. have y When the interest of your prospective client is directly adverse/contrary to the current client 1.7(a)(1) or 2. there is a significant risk that the representation of one or more clients will be materially limited by the lawyer's 1.7(a)(2), responsibilities to another client, a former client or a third person or by a personal interest of the lawyer. then you  Materially Limited dont go to y The critical questions are the likelihood that a difference in interests will eventuate and, if it does, 1.7(b). whether it will materially interfere with the lawyers independent professional judgment in BUT, go to considering alternatives or foreclose courses of action that reasonably should be pursued on 1.9 and do behalf of the client. an analysis y The question is whether the lawyer can effectively argue both sides of the same legal question without compromising interests of one client or the other y argue both sides of the same legal question without compromising interests of one client or the other b. Notwithstanding the existence of a concurrent conflict of interest under paragraph (a), a lawyer may represent a client if: 1. the lawyer reasonably believes that the lawyer will be able to provide competent and diligent representation to each Elements of affected client; AND (OBJECTIVE VIEW) Consent: Must 2. the representation is not prohibited by law; (IE: representing 2 criminal s in the same trial) AND satisfy first 3 3. the representation does not involve the assertion of a claim by one client against another client represented by the to get to #4, lawyer in the same litigation or other proceeding before a tribunal; AND all 4 must be 4. each affected client gives informed consent, confirmed in writing. (or the attorney memorializes the consent in a met. letter that client doesnt sign)  You will usually never get this b/c you will have to reveal too much confidential information NOTES - What is a conflict? o Anything that affects the lawyers duties to the client  Duties

y Loyalty y Confidentiality PUBLIC POLICY ISSUES y Diligence y Competence y Honesty y Independent professional judgment Loyalty is an essential element in the relationship b/w a lawyer and client. The lawyers professional judgment must be exercised solely for the benefit of the client, free of compromising influences and loyalties EVALUATING CONCURRENT CONFLICTS o Identify the client(s) o Identify the presence of the conflict (1.7(a); 1.8; 1.10) o If no conflict of interest, proceed with representation o If theres a conflict of interest then:  Ascertain consentability (1.7(b); 1.8; 1.10) y If not consentable decline representation y If consentable consult and secure informed consent confirmed in writing Consequences of Conflict of Interest o Declining representation o Withdrawal o Disqualification o Disgorgement of fees o Malpractice o Loss of future clientele o Negative reputation in the community sense o disloyalty and betrayal on the part of clients If a conflict of interest is apparent BEFORE a lawyer takes on a clients matter, the lawyer must not take it on. If a conflict becomes apparent only AFTER the lawyer has taken on the clients matter, and if informed consent of the affected clients will not solve the problem, then the lawyer must withdrawal (1.16(a)). (Screening does work for current clients) o A lawyers failure to handle a conflict properly can have three unpleasant consequences: disqualification as counsel in a litigated matter; professional discipline, and civil liability for legal malpractice. Comment 24: o Ordinarily a lawyer may take inconsistent legal positions in different tribunals at different times on behalf of different clients. There mere fact that advocating a legal position on behalf of one client might create precedent adverse to the interests of a client represented by the lawyer in an unrelated matter does not create a conflict. A conflict exists if there is a significant risk that a lawyers action on behalf of one client will materially limit the lawyers effectiveness in representing another client in a different case. Imputed Conflicts of Interest o Generally, lawyer who practice in a firm are treated as a single unit for conflict on interest purposes. That is, when one of the lawyers cannot take the matter b/c of a conflict of interest, the other lawyers in the firm are also barred from taking on that matter (1.10(a))  Meaning of Firm y The term firm includes not only an ordinary private law firm, but also other groups of lawyers who practice closely together, such as lawyers in a corporate law department, legal aid office, or prosecutors in a public defenders office y Factors to look at whether a group of lawyers should be regarded as a firm for conflict ofinterest purposes depends on many factors include, do the lawyers have a formal agreement among themselves, do they hold themselves out in a way that would make the public think they practice together as a firm, do they share revenues and responsibilities, do they have physical access to each other files, do they routinely talk among themselves about the matters they are handling

Rule 1.8 Conflict Of Interest: Current Clients: Specific Rules a. Business Transaction: A lawyer shall not enter into a business transaction with a client or knowingly acquire an ownership, possessory, security or other pecuniary interest adverse to a client unless: 1. The terms of the business transaction are fair to the client and are fully disclosed and transmitted in writing in a manner that can be reasonably understood by the client;  It must cover essential terms of the transaction and the lawyers role in the transaction y I.E: whether the lawyer is acting on behalf of the client

2. the client is advised in writing of the desirability of seeking and is given a reasonable opportunity to seek the advice of independent legal counsel on the transaction; and 3. the client gives informed consent, in a writing signed by the client, to the essential terms of the transaction and the lawyer's role in the transaction, including whether the lawyer is representing the client in the transaction.  Pecuniary Interest = making a loan to a client or taking a percentage of the business deal  Make sure not to give legal advice in reference to the business deal b/c it could be construed as representing the business transaction b. Adverse Use of Client Information: A lawyer shall not use information relating to representation of a client to the disadvantage of the client unless the client gives informed consent, except as permitted or required by these Rules. o The same rule applies to misuse of a FORMER OR PROSECTIVE clients confidential information.  Need informed consent unless required or allowed by the MRPC y 1.6(a) and (b) y 1.9(c): generally known y 4.1(b): shall not knowingly fail to disclose a material fact when it is necessary to avoid assisting a criminal act y 3.3 candor to the tribunal y 8.1 Bar admission and Disciplinary committee y 8.3: Reporting professional misconduct and y 1.14: diminished client c. Client Gifts and Gift Instruments **Not Consentable** o A lawyer shall not solicit any substantial gift from a client, including a testamentary gift, or prepare on behalf of a client an instrument giving the lawyer or o A person related to the lawyer any substantial gift unless the lawyer or other recipient of the gift is related to the client. o Related persons include a spouse, child, grandchild, parent, grandparent or other relative or individual with whom the lawyer or the client maintains a close, familial relationship.  In order to protect yourself, ask another person to draft the will  Lawyer may suggest self as executor. y Comment 8: The lawyer can appoint someone that they know be the executor, but the lawyer must pass through 1.7 to make sure there is not a conflict y Soliciting a Substantial Gift o A lawyer must not solicit a substantial gift from a client who is not the lawyers relative. However, a lawyer may accept a small gift from a client, such as a token of appreciation or an appropriate holiday gift. The rule does not prohibit a lawyer from accepting even a substantial gift, although the gift may be voidable for undue influence d. Media Rights: Prior to the conclusion of representation of a client, a lawyer shall not make or negotiate an agreement giving the lawyer literary or media rights to a portrayal or account based in substantial part on information relating to the representation. o However, a lawyer may acquire such rights after the clients legal matter is entirely completed, appeals and all o The rue is meant to protect the clients interest in effective representation from the conflict of the lawyers interest in maximizing the value of the literary or media rights.  The interest must actually relate to the representation to be prohibited, it is not enough that it deals with the same subject matter o Any disclosure of 1.6 information for this purpose would require client consent e. Financing Assistance to the Client: A lawyer shall not provide financial assistance to a client in connection with pending or contemplated litigation, except that: 1. a lawyer may advance court costs and expenses of litigation, the repayment of which may be contingent on the outcome of the matter; and  The purpose of this rule is to prevent a lawyer from gaining a large financial stake in the case and thus take away objective legal advice y I.E: Lawyer cant pay living expenses or doctor bills 2. a lawyer representing an indigent client may pay court costs and expenses of litigation on behalf of the client. f. Third Party Compensation: A lawyer shall not accept compensation for representing a client from one other than the client unless: 1. the client gives informed consent; 2. there is no interference with the lawyer's independence of professional judgment or with the client-lawyer relationship; and 3. information relating to representation of a client is protected as required by Rule 1.6.

10

NOTES y Always use 5.4(c) to mention the lawyer not being influenced y When an insurance company retains an attorney to defend an action against one of its insured, the insured is the client o The fact that the attorney also represents the insurer in no way alters his obligations or responsibilities to the insured y Ordinarily, since the interests of the insurer and insured are harmonious, there is no conflict and the attorney is able to exercise independent judgment for both clients o Therefore, in the usual instance, there is nothing improper or unethical about representing the interests of both y Other Third Party Payments o In juvenile cases, parents pay for childs attorney o In drug and other organized crime cases, when Mr. Big pays for Mr. Littles attorney o In labor relations cases, when the union pays for the union members attorney y Tripartite Relationship: The 3-way relationship: the insured, insurer and attorney. To modify the K duties to the insured, lawyer must rep under a reservation of rights or get a declaratory judgment g. Aggregate Settlements: A lawyer who represents two or more clients shall not participate in making an aggregate settlement of the claims of or against the clients, or in a criminal case an aggregated agreement as to guilty or nolo contendere pleas, unless each client gives informed consent, in a writing signed by the client. The lawyer's disclosure shall include the existence and nature of all the claims or pleas involved and of the participation of each person in the settlement. o NOTES  A lawyer may enter into an aggregate settlement agreement if all of the following are met: y The lawyer must assure that the clients have come to an agreement among themselves about how the aggregate sum will be shared (who ill pay how much and who will receive how much) y The lawyer must disclose to each of the clients all of the terms of the settlement including amount t be aid or received, the existence and nature of claims and defenses, the details regarding every other clients and what they will get, and how the fees and costs will be paid and to whom  These extensive disclosures require the lawyer to share confidential information with others, so at the outset they need to get the clients informed consent to do that. h. Limitations of Liability Agreements:A lawyer shall not: 1. make an agreement prospectively limiting the lawyer's liability to a client for malpractice unless the client is independently represented in making the agreement; or (Future) 2. settle a claim or potential claim for such liability with an unrepresented client or former client unless that person is advised in writing of the desirability of seeking and is given a reasonable opportunity to seek the advice of independent legal counsel (existing) i. Proprietary Interest in Subject Matter of Litigation:A lawyer shall not acquire a proprietary interest in the cause of action or subject matter of litigation the lawyer is conducting for a client, except that the lawyer may: 1. acquire a lien authorized by law to secure the lawyer's fee or expenses; and 2. contract with a client for a reasonable contingent fee in a civil case. j. Sexual Relations with a Client:No sex with a client unless you had sexual relations prior to the representation k. Disqualifications Attributed to Other Law Firm Members: If there is a conflict, it is imputed to the rest of the firm, the exception is j (sexual relations with a client) does not prevent others in the firm from representing them o NOTES  See also 3.7(b) Rule 1.9 Duties To Former Clients a. Former Client of Lawyer: A lawyer who has formerly represented a client shall not represent another person in the same or a substantially related matter in which that person's interests are materially adverse to the interests of the former client unless the former client gives informed consent, confirmed in writing. o NOTES  Former client of lawyer: Adversity + Same or substantially related matter = Written, informed consent from former client  It is not just the conflicts that are in front of you but what may become of it in the future. y Meaning of Substantially Related Matter The question is: Would it
be likely that youll have confidential information, not DO YOU have confidential info.

11

This depends on the facts of the particular situation. When a lawyer has been directly involved in a specific transaction, the lawyer cannot later oppose the former client in a dispute concerning the same transaction absent informed, written consent. o On the other hand, if the lawyer routinely handled a type of problem for a former client, the lawyer may oppose that former client in a wholly different problem of the same general type. b. Client of Layers Former Firm: A lawyer shall not knowingly represent a person in the same or a substantially related matter in which a firm with which the lawyer formerly was associated had previously represented a client 1. Whose interests are materially adverse to that person; and 2. About whom the lawyer had acquired information protected by Rules 1.6 and 1.9(c) that is material to the matter; unless the former client gives informed consent, confirmed in writing. o NOTES  Client of lawyers former firm: Adversity + same or substantially related matter + confidential information of client of former firm = former client conflict of interest and requires former clients informed consent confirmed in writing c. A lawyer who has formerly represented a client in a matter or whose present or former firm has formerly represented a client UNLESS: 1. The information has become generally known; or 2. Rules allow or require. NOTES - Even if former client consents, must go back to 1.7 and analyze to assure protection of current client o Remember to look at all sides of the conflict to ascertain whether duties to all clients prospective, current, former, organizational, individual, will be honored - Definitions o Same or Substantially Related Matter: same transaction or legal dispute, or  There is a substantial risk the Confidential factual (not legal) info that would normally have been obtained in the prior representation would materially advance the subsequent clients position, or o Common Facts: the test is not what Confidential info the lawyer actually has but what the lawyer would be expected to have o Matter: anything that is the subject of representation o Rule 1.10 Imputation Of Conflicts Of Interest: General Rule a. While lawyers are associated in a firm, none of them shall knowingly represent a client when any one of them practicing alone would be prohibited from doing so by Rules 1.7 or 1.9, unless 1. the prohibition is based on a personal interest of the prohibited lawyer and does not present a significant risk of materially limiting the representation of the client by the remaining lawyers in the firm; or 2. the prohibition is based upon Rule 1.9(a) or (b) and arises out of the disqualified lawyers association with a prior firm, and i. the disqualified lawyer is timely screened from any participation in the matter and is apportioned no part of the fee therefrom; ii. written notice is promptly given to any affected former client to enable the former client to ascertain compliance with the provisions of this Rule, which shall include a description of the screening procedures employed; a statement of the firm's and of the screened lawyer's compliance with these Rules; a statement that review may be available before a tribunal; and an agreement by the firm to respond promptly to any written inquiries or objections by the former client about the screening procedures; and iii. certifications of compliance with these Rules and with the screening procedures are provided to the former client by the screened lawyer and by a partner of the firm, at reasonable intervals upon the former client's written request and upon termination of the screening procedures. b. When a lawyer leaves a firm, the firm may represent a client against the client of a former firm member as long as: 1. No concurrent client conflict 2. It is not the same or substantially related to the matter that the former firm member represented the adverse client in, and 3. No on in the firm has 1.6 or 4. 1.9(c) information (confidential info) that is material to the matter. c. A disqualification prescribed by this rule may be waived by the affected client under the conditions stated in Rule 1.7. d. The disqualification of lawyers associated in a firm with former or current government lawyers is governed by Rule 1.11. NOTES - 1.10(a): 12

Applies to firms, co-employees of organizations and also space sharers where there are not firewalls (physical barriers) o Exceptions to Imputed Disqualification:  Family relationship: if you are married to your opponent, one of the associates in our firm can take over for the disqualified lawyer  Lawyer as a Witness: if you are a witness, then another lawyer in your firm can advocate  Sex: if you have sex with a client, you cannot represent them, but another associate can Proposed Model 1.10 (Screening) o Lawyers associated in a firm share each others 1.7 and 1.9 conflicts unless its a personal conflict of a lawyer and does not present a significant risk of materially limiting the representation; or o Its a 1.9(a) or (b) conflict and the disqualified lawyer is timely screened, doesnt share in the fee; written notice is promptly provided to all affected clients, and o Certification of compliance are provided to the former client o Under the proposed 1.10 screening provides a solution as long as it is timely o Under the old 1.10, screening may be used by a firm to gain the former clients consent to an otherwise consentable conflict under 1.9(b). o

Rule 1.11 Special Conflicts Of Interest For Former And Current Government Officers And Employees a. When a lawyer serves as an officer or employee of a government for a period and then leaves to enter private law practice, the government has a right to expect that its confidential information will not be abused. A lawyer who has formerly served as a public officer or employee of the government: 1. is subject to Rule 1.9(c); and 2. shall not otherwise represent a client in connection with a matter in which the lawyer participated personally and substantially as a public officer or employee, unless the appropriate government agency gives its informed consent, confirmed in writing, to the representation. b. When a lawyer is disqualified from representation under paragraph (a), no lawyer in a firm with which that lawyer is associated may knowingly undertake or continue representation in such a matter unless: 1. the disqualified lawyer is timely screened from any participation in the matter and is apportioned no part of the fee therefrom; and 2. written notice is promptly given to the appropriate government agency to enable it to ascertain compliance with the provisions of this rule.  Consent is not required for partners to rep. only if former gov. employee wants to represent c. Except as law may otherwise expressly permit, a lawyer having information that the lawyer knows is confidential government information about a person acquired when the lawyer was a public officer or employee, may not represent a private client whose interests are adverse to that person in a matter in which the information could be used to the material disadvantage of that person. o The term "confidential government information" means  Information that has been obtained under governmental authority and  Which, at the time this Rule is applied, the government is prohibited by law from disclosing to the public or has a legal privilege not to disclose and  Which is not otherwise available to the public. o Lawyers in Former Government Employees firm can still represent if FGE is timely screened and gets no fees nd remember FGE cant disclose the confidential go info to them except as 1.9(c) allows. d. Except as law may otherwise expressly permit, a lawyer currently serving as a public officer or employee: 1. is subject to Rules 1.7 and 1.9; and 2. shall not: I. participate in a matter in which the lawyer participated personally and substantially while in private practice or nongovernmental employment, unless the appropriate government agency gives its informed consent, confirmed in writing; or II. negotiate for private employment with any person who is involved as a party or as lawyer for a party in a matter in which the lawyer is participating personally and substantially, except that a lawyer serving as a law clerk to a judge, other adjudicative officer or arbitrator may negotiate for private employment as permitted by Rule 1.12(b) and subject to the conditions stated in Rule 1.12(b). e. As used in this Rule, the term "matter" includes: 1. any judicial or other proceeding, application, request for a ruling or other determination, contract, claim, controversy, investigation, charge, accusation, arrest or other particular matter involving a specific party or parties, and 2. any other matter covered by the conflict of interest rules of the appropriate government agency. - NOTES 13

o o o

Matteris any judicial or other proceeding, application, request ofr a ruling of other determination, contract, claim, controversy, investigation, charge, accusation, arrest or other particular matter involving specific party of parties, and  Any other matter covered by the COI rules of the appropriate government agency (1.11(e)) Personally: direct involvement through decision, approval, disapproval, recommendation, advice, investigation or otherwise. Incldessbordinates and non-lawyers who becomes lawyers Substantially: significant or reasonable appearance of significant involvement; more than perfunctory; more than mere knowledge; Any Matter: no adversity required.

Rule 1.12 Former Judge, Arbitrator, Mediator Or Other Third-Party Neutral a. Officers cant represent parties in matters where previously participated personally and substantially as a judge w/o written informed consent from all parties b. No negotiation of employment with anyone involved as a party or lawyer in matters the adjudicative officer is participating personally and substantially as a judge or mediation. Law clerks are excepted with notice to the judge c. All lawyers in disqualified lawyers firm are disqualified unless the disqualified lawyer is screened, doesnt share in the fees and all parties are promptly served with written notice o NOTE: screening works for former judges too former government officials d. Partisan arbitrators and mediators are not subsequently prohibited from representing the party who selected them NOTES - What are Former Judicial Officers? Former judges, arbitrators, mediators or third party neutrals Rule 1.13 Organization As Client a. A lawyer employed or retained by an organization represents the organization acting through its duly authorized constituents. b. If a lawyer for an organization knows that an officer, employee or other person associated with the organization is engaged in action, intends to act or refuses to act in a matter related to the representation that is a violation of a legal obligation to the organization, or a violation of law that reasonably might be imputed to the organization, and that is likely to result in substantial injury to the organization, then the lawyer shall proceed as is reasonably necessary in the best interest of the organization. Unless the lawyer reasonably believes that it is not necessary in the best interest of the organization to do so, the lawyer shall refer the matter to higher authority in the organization, including, if warranted by the circumstances to the highest authority that can act on behalf of the organization as determined by applicable law. c. Except as provided in paragraph (d), if 1. despite the lawyer's efforts in accordance with paragraph (b) the highest authority that can act on behalf of the organization insists upon or fails to address in a timely and appropriate manner an action, or a refusal to act, that is clearly a violation of law, and 2. the lawyer reasonably believes that the violation is reasonably certain to result in substantial injury to the organization,then the lawyer may reveal information relating to the representation whether or not Rule 1.6 permits such disclosure, but only if and to the extent the lawyer reasonably believes necessary to prevent substantial injury to the organization. d. Paragraph (c) shall not apply with respect to information relating to a lawyer's representation of an organization to investigate an alleged violation of law, or to defend the organization or an officer, employee or other constituent associated with the organization against a claim arising out of an alleged violation of law. e. A lawyer who reasonably believes that he or she has been discharged because of the lawyer's actions taken pursuant to paragraphs (b) or (c), or who withdraws under circumstances that require or permit the lawyer to take action under either of those paragraphs, shall proceed as the lawyer reasonably believes necessary to assure that the organization's highest authority is informed of the lawyer's discharge or withdrawal. f. In dealing with an organization's directors, officers, employees, members, shareholders or other constituents, a lawyer shall explain the identity of the client when the lawyer knows or reasonably should know that the organization's interests are adverse to those of the constituents with whom the lawyer is dealing. g. A lawyer representing an organization may also represent any of its directors, officers, employees, members, shareholders or other constituents, subject to the provisions of Rule 1.7. If the organization's consent to the dual representation is required by Rule 1.7, the consent shall be given by an appropriate official of the organization other than the individual who is to be represented, or by the shareholders. NOTES - The organization lawyer must advise constituents that the lawyer represents the organization, not the constituents, if the attorney knows or reasonably should know that the constituents and the organizations have adverse interests - If the organization wants to fix a past crime, the lawyer cannot report out because the corp is trying to fix it internally

14

Rule 1.14 Client With Diminished Capacity a. When a client's capacity to make adequately considered decisions in connection with a representation is diminished, whether because of minority, mental impairment or for some other reason, the lawyer shall, as far as reasonably possible, maintain a normal client-lawyer relationship with the client. b. When the lawyer reasonably believes that the client has diminished capacity, is at risk of substantial physical, financial or other harm unless action is taken and cannot adequately act in the client's own interest, the lawyer may take reasonably necessary protective action, including consulting with individuals or entities that have the ability to take action to protect the client and, in appropriate cases, seeking the appointment of a guardian ad litem, conservator or guardian. c. Information relating to the representation of a client with diminished capacity is protected by Rule 1.6. When taking protective action pursuant to paragraph (b), the lawyer is impliedly authorized under Rule 1.6(a) to reveal information about the client, but only to the extent reasonably necessary to protect the client's interests.

Rule 1.16 Declining Or Terminating Representation a. Mandatory Withdrawal: A Lawyer must decline or withdraw from representation if the client demands that the lawyer engage in conduct that is illegal or violates the rules of professional conduct of other law; the lawyers physical or mental condition materially impairs the lawyers ability to represent the client; or the lawyer is discharged o When the lawyer was appointed to represent the client, withdrawal ordinarily requires approval of the appointing authority (6.2). If the client demands the lawyer engage in unprofessional conduct, lawyer asks for withdrawal, but this would violate attorney-client relationship is such information is disclosed. The lawyer normally just needs to say to the court that professional considerations require termination of the representation and that is sufficient. Lawyers should be mindful of their obligations to both clients and the court under 1.6 and 3.3 b. Permissive Withdrawal: a lawyer may withdraw from representing a client if it can be done without material adverse effect on the clients interests or if the client consents to withdraw. In addition, the attorney may withdraw despite an adverse impact on the clients interests in the situations listed below provided the circumstances are severe enough to justify harming the clients interests o Client persists in Criminal or Fraudulent Conduct  A lawyer may withdraw from representing a client if the client persists in a course of action that involves the lawyers services and that the lawyer reasonably believes is criminal or fraudulent. Note that if the clients criminal or fraudulent conduct involves some assistance by the lawyer, then the lawyer MUST withdraw o Client has used attorneys services to commit a past crime or fraud o Clients objective is repugnant or against the lawyers beliefs o Client Breaks promises to attorney  This allows a permissible withdraw from representing the client if the client substantially fails to fulfill and obligation to the attorney and the attorney warned him that he would withdraw if it was not fulfilled o Financial Hardship for attorney o Client will not Cooperate  If the client has made the lawyers work unreasonably difficult (i.e.: wont help attorney in discovery proceedings) o Other good cause  It is important to remember that even if the lawyer has been unfairly discharged by the client the lawyer must take all steps to mitigate any negative consequences that the client may have c. Withdrawal while in LitigationA lawyer must comply with applicable law requiring notice to or permission of a tribunal when terminating a representation. When ordered to do so by a tribunal, a lawyer shall continue representation notwithstanding good cause for terminating the representation. d. Duties upon withdrawal o Provide the client with reasonable notice of withdrawal, providing the client with time to obtain another attorney, refunding attorneys fees paid in advance and not yet earned and expense advances not yet spent, and returning all papers and property to which the client is entitled. You are allowed to retain copies. NOTES - A lawyer should not accept representation in a matter unless it can be performed competently, promptly, without improper conflict of interest, and to completion. Ordinarily, a representation in a matter is completed when the agreed-upon assistance has been concluded - Even a lawyer who withdraws from representation must protect the clients interest with reasonably practicable measures like: o Adequate notice to the client of the termination and upcoming deadlines o Returning of files and items to the client 15

o Reimbursing the client for unearned advance fees and expenses o Withdrawing lawyer may retain client-related papers as the law permits For every matter you do accept, write an engagement letter stating: o The scope of what you are going to do and not do for the client o The method for calculating attorney fees. o Have your client sign and return the letter For every matter you dont accept, write a non-engagement or declination letter stating: o We do NOT represent you o We are going to do nothing on your behalf o You should consult another attorney and take other action as soon as possible o The statute of limitations may be running on your claim Generally, the clients are free to fire their lawyers at any time, with or without cause o The power of the client to discharge his or her lawyer is an implied term of the retainer contract, so  The client does not breach the contract when he fires the lawyer based on a reasonable subjective dissatisfaction with the lawyers services, even if the client does not have good cause  The fact that a lawyer has been hired under a contingent fee contract does not affect the clients absolute right to discharge the lawyer Whether the lawyer can collect a fee depends on whether the firing is with or without cause o If the client fires the lawyer for serious misconduct such as fraud or illegal conduct the lawyer is not entitled to a fee o If the lawyer acts competently and there is no serious misconduct, but the client has a good faith basis to be dissatisfied with the lawyer is entitled to be compensated for work done prior to discharge  But in a contingency fee contract, the attorney must await the occurrence of the contingency. Because contingency fee agreements anticipate that no fee will be payable unless and until the contingency fee occurs Quantum Meruit Fees o Where a client has a good faith basis to terminate the lawyer-client relationship but there is no serious misconduct warranting of any fee o The lawyer is entitled to compensation based on the reasonable value of services rendered prior to discharge, considering as factors:  The reasonable value of the benefits the client obtained as a result of the services, and  The nature and gravity of the cause that led to the attorneys discharge The Charging Lien: a lawyers right to fees and costs expended on behalf of the client from a fund recovered by the lawyers efforts and also the right to have the court interfere to protect the fund from the clients creditors before the lawyer is paid o Charging liens are to protect lawyers against the dishonest client who tries to evade paying the lawyer after using the lawyers services to establish the clients claim Retaining Liens: are where the lawyer retains client property until the fee is paid. You cant do this where the property you want to retain is the client file and the client needs it to process the case Attorneys duties upon termination or representation o An attorney that withdraws from a matter must comply with local laws that require notice to or permission of the tribunal before withdrawal. Moreover, upon termination of the representation, the attorney must take reasonable steps to protect the clients interest, including  Providing the client with reasonable notice of the withdrawal, providing the client with time to obtain another attorney, refunding attorneys fees paid in advance and not yet earned and expense advances not yet spent, and returning all papers and property to which the client is entitled Declining/Terminating the Attorney Client Relationship o When must the lawyer terminate?  When rep will violate rules  When fired o When may the lawyer terminate?  When disagree/client is repugnant  Client may use your services in a criminal manner  Financial burden  Dont have the time  Not getting along w/client o When must the lawyer get court approval?  Pending action o What happens to the duty of due care by the lawyer to that client? 16

 Still exists Fees for terminated lawyers dependupon the circumstances  Fired for no cause: contract value (if its a valid K) or at least quantum meruit  Fired for cause: maybe quantum meruit but likelihood is no fees

1.18 Duties to Prospective Clients - A prospective client is a person who merely discusses with a lawyer the possibility of forming a client-lawyer relationship regarding a matter. The privilege covers preliminary communications leading up to an attorney-client relationship, even if no such relationship develops. This is the rule that gives duty of confidentiality to a prospective. - An attorney who has met with a prospective client cannot represent someone with materially adverse interest in the same or a substantially related matter if: o The attorney obtained potentially harmful info, except as allowed under 1.18(d) o And also provides that other attorneys in the firm cannot represent a client with adverse interests unless allowed under 1.18(d) - When the lawyer has received disqualifying info, representation is permissible if: o Both the affected client and the prospective client have given informed consent, confirmed in writing, or; o The lawyer who received the info took reasonable measures to avoid exposure to more disqualifying info than was reasonably necessary to determine whether to represent the prospective client; and  The disqualified lawyer is timely screened from any participation in the matter and is apportioned no part of the fee; and  Written notice is promptly given to the prospective client - Remember that conflicts of interest apply to prospective clients, too, so a lawyer can be disqualified on the basis of confidential information gained from a prospective client and the conflict in imputed to the rest of the firm unless - BOTH the former (prospective) client and the current client consents or the lawyer avoids disqualifying information, is screened from the new representation and the former (prospective) client is provided

COUNSELOR Rule 2.1 Advisor - In representing a client, a lawyer shall exercise independent professional judgment and render candid advice. In rendering advice, a lawyer may refer not only to law but to other considerations such as moral, economic, social and political factors, that may be relevant to the client's situation NOTES - Lawyer may also encourage a client to seek advice from persons in related professions such as accountants, psychiatrist, physician, or family counselor - You should give advice to a client when doing so appears to be in the clients interest. - The client deserves straightforward advice expressing the lawyers honest assessment - A lawyers advice can often concern unpleasant facts and alternatives the client might wish to avoid - A lawyer should try to preserve the clients morale by framing her advice in as acceptable a form as honestly permits - But in the end, the lawyer should give candid advice even when it will be unpalatable to the client

Rule 2.4 Third Party Neutral a. A lawyer serves as a third-party neutral when the lawyer assists two or more persons who are not clients of the lawyer to reach a resolution of a dispute or other matter that has arisen between them. Service as a third-party neutral may include service as an arbitrator, a mediator or in such other capacity as will enable the lawyer to assist the parties to resolve the matter. b. A lawyer serving as a third-party neutral shall inform unrepresented parties that the lawyer is not representing them. When the lawyer knows or reasonably should know that a party does not understand the lawyer's role in the matter, the lawyer shall explain the difference between the lawyer's role as a third-party neutral and a lawyer's role as one who represents a client.

ADVOCATE Rule 3.1 Meritorious Claims And Contentions

17

A lawyer shall not bring or defend a proceeding, or assert or controvert an issue therein, unless there is a basis in law and fact for doing so that is not frivolous, which includes a good faith argument for an extension, modification or reversal of existing law. A lawyer for the defendant in a criminal proceeding, or the respondent in a proceeding that could result in incarceration, may nevertheless so defend the proceeding as to require that every element of the case be established.

NOTES - Criminal defense lawyers are specially allowed to defend cases by requiring that every element of the case be established by the prosecution - This rule relates to 1.2 in that lawyer cannot violate the MR or assist a client in fraud or criminal behavior. A lawyer may discuss the penalties with the client if they chose to do that - SLAP suit Stop Lawsuit Abuse Plaintiffs doesnt matter if initial suit was warranted, the party that I sued is trying to prevent them from filing suit in future o Elements:  Lack of probable cause  Improper purpose - Threatening Criminal Prosecution o Old Code: lawyers prohibited from threatening criminal prosecution to get an advantage over an opposing arty o MR: no explicitly prohibition against threat of criminal actions  BUTa particular threat might still violate: y 3.1 (frivolous claim) No basis for threat of criminal prosecution y 4.1 untruthful statements) Not really going to prosecute y 4.4 (actions to harass) harassment y 8.4(b)(criminal acts reflecting on lawyers fitness) extortion y 8.4(d)(conduct prejudicial to admin of justice) catch all y 8.4(e) - A threat to bring criminal charges for the purpose of advancing a civil claim would violate MR: o If the criminal wrongdoing is unrelated to the clients civil claim; o If the lawyer does not believe both the civil clam and the potential criminal charges are well founded; o Of if the threat constitutes an attempt to exert or suggest improper influence over the criminal process **Otherwise the threat is theoretically ethically permissible** Rule 3.3 Candor Toward The Tribunal (Balancing the lawyers duty to the client with the lawyers duty to protect the integrity of the judicial system) a. A lawyer shall not knowingly: 1. Make a false statement of fact or law to a tribunal or fail to correct a false statement of material fact or law previously made to the tribunal by the lawyer; 2. an attorney is subject to discipline for knowingly failing to disclose to the court a legal authority in the controlling jurisdiction that is directly adverse to the clients position and that has not been disclosed by opposing counsel. Dont need to cite authority from non-controlling jurisdictions.  EX: this is a case that may kill your clients case. If you find a case that is not good for your case and your opponent hasnt cited that case, you must disclose the case to the court and other side.  You can still try to argue around this case  If you didnt disclose during the trial, you would still lose on appeal anyways 3. You cannot offer evidence that the lawyer knows to be false. If a lawyer, the lawyers client, or a witness called by the lawyer, has offered material evidence and the lawyer comes to know of its falsity, the lawyer shall take reasonable remedial measures, including, if necessary, disclosure to the tribunal. A lawyer may refuse to offer evidence, other than the testimony of a defendant in a criminal matter, that the lawyer reasonably believes is false.  If you reasonably believe that it may be false, you cant use it  No Obligation To Volunteer Harmful Facts y An attorney generally has no obligation to volunteer a fact that is harmful to his clients case. The adversary system assumes that opposing sides can use discovery proceedings and their own investigations to find out the facts. If an attorneys adversary fails to uncover a harmful fact, an injustice may result, but that is simply the way the adversary system works b. A lawyer who represents a client in an adjudicative proceeding and who knows that a person intends to engage is engaging or has engaged in criminal or fraudulent conduct related to the proceeding shall take reasonable remedial measures, including, if necessary, disclosure to the tribunal. o Discuss is ALWAYS the first step. You are there to counsel and, at all times to protect the client, sometimes even from himself.  Discuss:

18

Potential consequences of perjury Plans to overcome negative inferences from failure to testify The lawyers obligations under the MR and the law tell the client if he lies to the court you will have to tell the court o Then you can try to withdraw by telling the court that there is a conflict, but be very general o You dont want to tell the court unless he has actually done it b/c he might change his mind o You will petition the court to let the testify in the narrative o If allowed will give testimony in the narrative then the prosecutor gets to cross examine the s attorney cannot object o s attorney cannot talk about s testimony in closing o If the fails to tell the court about the perjury, then the attorney must o Prevent Corruption for the Proceedings (3.3b)  Examples of such conduct are hiding or destroying evidence, bribing a witness, intimidating jury, buying a judge, and failing to obey a law or court order to disclose information. Appropriate measures include disclosure to the court if that becomes necessary c. MR 3.3 trumps 1.6 (confidentiality and A/C Privilege) so you MUST disclose even if it involves revealing confidential information. o You have the duty of candor to the court to the conclusion of the proceeding  The acquittal of a criminal  The conclusion of the appeal or expiration of the time for filing an appeal in all other matters d. Exception: Ex Parte Proceedings o In this type of proceeding only one side is present. At the hearing the lawyer for the party that is being heard alone must inform the court of all the material facts, both helpful and harmful, that bear on the issue before the court. This is so that the court may make an informed decision without the other party present NOTES - Using False Evidence 3.3 o In a matter pending before a tribunal, a lawyer is subject to discipline for offering evidence that the lawyer knows is false. ABA 3.3 knows means actual knowledge, but actual knowledge can be inferred from the circumstances. A lawyer should resolve doubts about veracity in favor of her client, but a lawyer cannot ignore an obvious falsehood o A lawyer may refuse to offer evidence that she reasonably believes is false, except for a criminal on his own behalf.  Discovery of falsity after evidence has been offered y If the lawyer has offered a piece of evidence and later discovers that it is false, she must take reasonable remedial measures. o First, the lawyer must speak confidentially with her client, urging the clients cooperation in withdrawing or correcting the false evidence o Second, if the client will not cooperate, the lawyer should consider asking the courts permission o withdraw. Ordinarily the withdrawal is not mandatory, but if it creates a rift /w the client and attorney that he can no longer represent them effectively he may.  Withdrawal alone is not a sufficient remedial step if it leaves the false evidence before the tribunal, the lawyer needs to make a motion to strike the false evidence or take other steps to cancel its effect o Thirdly, if withdrawal is not permitted or will not solve the problem, the lawyer must disclose the situation to the judge, even if it means disclosing the clients information that would otherwise be protected under the duty of confidentiality  The criminal defense has a sixth amendment right to counsel and a constitutional right to testify on his own behalf. When the criminal defense attorney has a client that insists on testifying to something that the lawyer knows is false he must do the following: y Witnesses in either criminal or civil cases and civil s and s are not constitutionally entitled to testify o If you reasonably believe that the witness will lie, dont use them y You must give the client the benefit of the doubt so you either have to know or believe, with a firm factual basis (knowledge or believes beyond a reasonable doubt), that the client will perjure himself before you take steps to avoid o If you think they will lie, but dont know for sure, you can move forward y y y

19

If you dont know that he is lying then you put him on the stand o The Majority: try to convince the client not to testify falsely, if the insists on testifying, the lawyer should consider withdrawal, fi this is not permitted, they must reveal the situation to the judge even if it means disclosing the information o The Minority: ask them to testify in the narrative fashion. The defense asks the question that calls on a narrative answer such as What happened? The defense is not allowed to rely on the false parts of the story when arguing the case to the Trier of Fact. 3.3 defers to the local law in jurisdictions that follow the minority view.

Rule 3.8 Special Responsibilities Of A Prosecutor - The prosecutor in a criminal case is not simply an advocate but also a minister of justice; the prosecutors primary goals are to seek justice, not to convict. Thus, the prosecutor must assure that the is tried by fair procedures and that guilt is decided on proper and sufficient evidence. Local laws may impose additional duties on a prosecutor, and failure to comply with such laws is grounds for professional discipline - 3.8(a): Prosecuting without probable cause o A prosecutor must not prosecute a charge that she knows is not supported by probable cause - 3.8(b): Prosecuting Accused right to Counsel o A prosecutor must make reasonable efforts to assure that the accused is advised of his right to counsel, advised of the procedures for obtaining counsel, and given a reasonable opportunity to obtain counsel. - 3.8(c): Securing Waiver of Pre-trial Rights o A prosecutor must not seek to obtain from an unrepresented accused a waiver of important pre-trial rights, such as the right to a preliminary hearing  I.E.: a prosecutor must not ask a who has invoked right to counsel to clear this up and avoid all of the legal technicalities - 3.8(d): Disclosing Evidence that may Help Defense o A prosecutor must timely disclose to the defense all evidence and information known to him that may negate the guilt of the accused or mitigate the degree of the offense. Failure to disclose such information may deprive the of due process - 3.8(e): Subpoenaing Other Lawyers o A prosecutor must not subpoena another lawyer to give evidence about a client or former client unless the evidence is not privileged, is essential, and cannot be obtained in another way - 3.8(f): Public statements about pending matters o Except in statements necessary to inform the public of the nature and extent of the prosecutors action and serve a legitimate law enforcement purpose, a prosecutor must not make extrajudicial statements that have a substantial likelihood of heightening public condemnation of the accused. A prosecutor must take reasonable care to prevent investigators, police, employees, and other subordinates from making such statements - These rules are also incumbent on all government lawyers, not merely public prosecutors. A government lawyer must also not litigate a civil matter or continue a matter that is obviously unfair. - 3.8(g): New Credible Information After the has been convicted o If prosecutor learns of new credible material evidence that shows a convicted did not commit the offense, he shall promptly disclose it to the appropriate court, and if the conviction was obtained in the prosecutors court, he needs to tell the of the evidence, if he proves that there is clear and convincing evidence that the did not commit the crime, the prosecutor shall seek to remedy the conviction. TRANSACTIONS WITH PERSONS OTHER THAN CLIENTS Rule 4.1 Truthfulness In Statements To Others In the course of representing a client a lawyer shall not knowingly: a. make a false statement of material fact or law to a third person; or b. fail to disclose a material fact to a third person when disclosure is necessary to avoid assisting a criminal or fraudulent act by a client, unless disclosure is prohibited by Rule 1.6. (if 1.6(b) is happening a lawyer must disclose under this rule) NOTES - Generally, a lawyer has no duty to inform a third person of relevant facts. However, a lawyer must not misrepresent the facts o Types of Misrepresentations  Makes a statement knowing that it is false, when the lawyer states something that is partly true but misleading, or in some contexts when the lawyer fails to speak or act o Distinguish Conventional Puffery

20

Under generally accepted conventions in negotiations, certain types of statements ordinarily are not taken as statements of material fact y Estimates of price or value placed on the subject of the transaction are ordinarily regarded as mere puffery, and so is a statement of partys intentions as to settlement of a claim 4.1(b) only trumps 1.6(b)(2) and (3) if theyre applicable in this situation, then it turns into a must rule. Make sure to check that the client used the attorneys services to commit the fraud Where the duty of confidentiality prevents the lawyer from disclosing material facts, and where continued representations would require the lawyer to assist in the clients crime or fraud, the lawyer must withdraw 1.6(a), 1.2(d). The lawyer may notify the affected third person of the withdrawal and may withdraw or disaffirm any opinion, documents, or affirmation previously furnished in connection with the matter

Rule 4.2 Communication With Person Represented By Counsel - In representing a client, a lawyer shall not communicate about the subject of the representation with a person the lawyer knows to be represented by another lawyer in the matter, unless the lawyer has the consent of the other lawyer or is authorized to do so by law or a court order. o Corporations and other organizations are persons for purposes of this rule. Thus, a lawyer must get the consent of the organizations counsel before communicating with the following constituents. Consent is not needed before talking to a former constituent of the organizations o Parties to a matter may communicate directly with each other, and a lawyer is prohibited from advising a client concerning communication that the client is legally entitled to make o Communications authorized by law may include communications by a lawyer on behalf of a client who is exercising a constitutional or legal right to communicate with the government o Opposing counsel is prohibited from contacting organizational constituents who:  Facilitates legal advice  Can bind or commit the organization legally  Whose act or non-actions can be imputed to the organization y Unless the organizational lawyer consents or is present Rule 4.3 Dealing With Unrepresented Person - In dealing on behalf of a client with a person who is not represented by counsel, a lawyer shall not state or imply that the lawyer is disinterested. - When the lawyer knows or reasonably should know that the unrepresented person misunderstands the lawyers role in the matter, the lawyer shall make reasonable efforts to correct the misunderstanding. - The lawyer shall not give legal advice to an unrepresented person, other than the advice to secure counsel, if the lawyer knows or reasonably should know that the interests of such a person are or have a reasonable possibility of being in conflict with the interests of the client. Rule 4.4 Respect For Rights Of Third Persons a. Bans conduct that has no substantial purpose other than to embarrass, delay, or burden someone & evidence gathering mechanisms that violate someones rights b. Lawyers who receive documents or correspondence inadvertently sent must tell the sender o The MR do not address some related questions regarding what to do with the document; send it back or if the disclosure acts as a waiver LAW FIRMS AND ASSOCIATIONS Rule 5.1 Responsibilities Of Partners,Managers, And Supervisory Lawyers a. The partners or managers ina firm must make reasonable efforts to ensure that the lawyer working for the firm adhere to the rules of professional conduct. This is done by continuing education programs in legal ethics, and some firms have a designated partner or committee to whom a junior lawyer may turn in confidence for assistance on an ethics issue b. A lawyer with direct supervisory authority over another lawyer: must make similar efforts to ensure that the attorney complies with the MR c. Ethical responsibilities for another lawyers misconduct 1. A lawyer is subject to discipline for a disciplinary violation committed by a second lawyer if, the first lawyer ordered the second lawyers misconduct of knew about it and ratified it; or 2. The first lawyer is a partner or a manager or has direct supervisory responsibility over the second lawyer, and she knows about the misconduct at a time when the consequences can be avoided or mitigated and fails to take reasonable remedial action

21

Rule 5.2 Responsibilities Of A Subordinate Lawyer a. Orders from a supervisory lawyer are no excuse for clearly unethical conduct a lawyer must follow the rules even when he is acting under the direction of another person o The subordinate lawyer does however need to have the knowledge that is required for some ethical obligation b. A subordinate lawyer does not violate the rules of professional conduct by acting in accordance with a supervisors reasonable resolution of an arguable question of professional duty o When a debatable ethics question arises, someone must decide on a course of action, and that responsibility must rest with the supervisory lawyer. The subordinate lawyer will not be subject to discipline for doing what the supervisor directed.

Rule 5.3 Responsibilities Regarding Nonlawyer Assistants With respect to a nonlawyer employed or retained by or associated with a lawyer: a. a partner, and a lawyer who individually or together with other lawyers possesses comparable managerial authority in a law firm shall make reasonable efforts to ensure that the firm has in effect measures giving reasonable assurance that the person's conduct is compatible with the professional obligations of the lawyer; b. a lawyer having direct supervisory authority over the nonlawyer shall make reasonable efforts to ensure that the person's conduct is compatible with the professional obligations of the lawyer; and c. a lawyer shall be responsible for conduct of such a person that would be a violation of the Rules of Professional Conduct if engaged in by a lawyer if: 1. the lawyer orders or, with the knowledge of the specific conduct, ratifies the conduct involved; or 2. the lawyer is a partner or has comparable managerial authority in the law firm in which the person is employed, or has direct supervisory authority over the person, and knows of the conduct at a time when its consequences can be avoided or mitigated but fails to take reasonable remedial action. NOTES - All people that work or are employed with a law firm need to be instructed regarding the ethics of the profession, and the lawyer should be ultimately responsible for their work. The partners or manager of the firm are responsible for making sure that the conduct of the non-lawyer is compatible with the obligations of the profession - The lawyer is subject to discipline from a non-lawyers conduct if they ordered the conduct or knew about it and ratified it, or if the lawyer knows about the misconduct at a time when its consequences can be avoided or mitigated and fails to take reasonable remedial action Rule 5.4 Professional Independence Of A Lawyer a. A lawyer or law firm shall not share legal fees with a nonlawyer, except that: 1. an agreement by a lawyer with the lawyer's firm, partner, or associate may provide for the payment of money, over a reasonable period of time after the lawyer's death, to the lawyer's estate or to one or more specified persons; 2. a lawyer who purchases the practice of a deceased, disabled, or disappeared lawyer may, pursuant to the provisions of Rule 1.17, pay to the estate or other representative of that lawyer the agreed-upon purchase price; 3. a lawyer or law firm may include nonlawyer employees in a compensation or retirement plan, even though the plan is based in whole or in part on a profit-sharing arrangement; and 4. a lawyer may share court-awarded legal fees with a nonprofit organization that employed, retained or recommended employment of the lawyer in the matter. b. A lawyer shall not form a partnership with a nonlawyer if any of the activities of the partnership consist of the practice of law. c. A lawyer shall not permit a person who recommends, employs, or pays the lawyer to render legal services for another to direct or regulate the lawyer's professional judgment in rendering such legal services. d. A lawyer shall not practice with or in the form of a professional corporation or association authorized to practice law for a profit, if: 1. a nonlawyer owns any interest therein, except that a fiduciary representative of the estate of a lawyer may hold the stock or interest of the lawyer for a reasonable time during administration; 2. a nonlawyer is a corporate director or officer thereof or occupies the position of similar responsibility in any form of association other than a corporation ; or 3. anonlawyer has the right to direct or control the professional judgment of a lawyer. NOTES

22

Rule 5.5 Unauthorized Practice Of Law; Multijurisdictional Practice Of Law a. A lawyer shall not practice law in a jurisdiction in violation of the regulation of the legal profession in that jurisdiction, or assist another in doing so. b. A lawyer who is not admitted to practice in this jurisdiction shall not: 1. except as authorized by these Rules or other law, establish an office or other systematic and continuous presence in this jurisdiction for the practice of law; or 2. hold out to the public or otherwise represent that the lawyer is admitted to practice law in this jurisdiction. c. A lawyer admitted in another United States jurisdiction, and not disbarred or suspended from practice in any jurisdiction, may provide legal services on a temporary basis in this jurisdiction that: 1. are undertaken in association with a lawyer who is admitted to practice in this jurisdiction and who actively participates in the matter; 2. are in or reasonably related to a pending or potential proceeding before a tribunal in this or another jurisdiction, if the lawyer, or a person the lawyer is assisting, is authorized by law or order to appear in such proceeding or reasonably expects to be so authorized; 3. are in or reasonably related to a pending or potential arbitration, mediation, or other alternative dispute resolution proceeding in this or another jurisdiction, if the services arise out of or are reasonably related to the lawyers practice in a jurisdiction in which the lawyer is admitted to practice and are not services for which the forum requires pro hac vice admission; or 4. are not within paragraphs (c)(2) or (c)(3) and arise out of or are reasonably related to the lawyers practice in a jurisdiction in which the lawyer is admitted to practice. d. A lawyer admitted in another United States jurisdiction, and not disbarred or suspended from practice in any jurisdiction, may provide legal services in this jurisdiction that: 1. are provided to the lawyers employer or its organizational affiliates and are not services for which the forum requires pro hac vice admission; or 2. are services that the lawyer is authorized to provide by federal law or other law of this jurisdiction. NOTES

PUBLIC SERVICE Rule 6.1 Voluntary Pro Bono Publico Service - Violation for not offering pro bono service is not grounds for discipline. Every lawyer should spend 50 hours of pro bono service for people who cannot pay - Ways to do this: o Free services to low income persons or organizations designed to serve them o Free or substantially reduced fee services to persons or community groups who would otherwise have to devote most of their income to lawyers fees o Contribute money toward legal services for the poor Rule 6.2 Accepting Appointments A lawyer shall not seek to avoid appointment by a tribunal to represent a person except for good cause, such as: a. representing the client is likely to result in violation of the Rules of Professional Conduct or other law; b. representing the client is likely to result in an unreasonable financial burden on the lawyer; or c. the client or the cause is so repugnant to the lawyer that the lawyers representation of the client would be imparied

INFORMATION ABOUT LEGAL SERVICES Rule 7.1 Communications Concerning A Lawyer's Services - A lawyer is subject to discipline for any type of communication about the lawyer or the lawyersservices that are false or misleading. This rule applies to all types of communications, including advertisements, personal communications, office signs, professional cards, professional announcements, letterheads, brochures, letters or emails, and recorded phone messages - Defines false or misleading to include statements materially misrepresenting facts,, and the omission facts needed to make the statement as a whole note materially misleading Rule 7.2 Advertising - Must be true and non-misleading. They may advertise in written, recorded, and electronic media, including public media

23

Must get consent of clients names if used in ads Every ad must include the name and office address of at least on lawyer or law firm that is responsible for its content payments for recommending the lawyers services o Except in connection with the sale of a law practice, a lawyer must not give anything of value to a person for recommending the lawyers services o Lawyers may be involved in a qualified referral service program as long as it is approved by the appropriate regulatory authority. Prohibits a lawyer from paying someone to recommend the lawyer, but allows: o Payment of standard advertising expense  Payment of fess to qualified legal services plans and lawyer referral services. A qualified lawyer referral service is a lawyer referral service that has been approved by appropriate regulatory authority; and  Payment for the purchase of a law practice Reciprocal Referral Agreements o This is allowed to be done with a non-lawyer professional as long as it is not exclusive, the referred client must be told about the arrangement, and if the arrangement creates a conflict of interest for either the referring or receiving lawyer then that lawyer must obtain the clients informed written consent under 1.7. The reciprocal agreement must not interfere with the lawyers professional judgment as to making referrals; it also should not be indefinite in duration and should be reviewed periodically to make sure they comply with the rules. Comment 6: A lawyer may pay the usual charges of a legal service plan or a not-for-profit or qualified lawyer referral service. A legal service plan is a prepaid legal service plan that assists perspective clients to secure legal representation.. A lawyer referral service is any organization that holds itself out to the public as a lawyer referral service

Rule 7.3 Direct Contact With Prospective Clients - The basic rules is that a lawyer must not seek fee paying work by initiating personal or live telephone contact, or any real time contact, with a non-lawyer prospect with whom the lawyer has no family, close personal relationship, or prior professional relationship - You may offer free legal advice even if the representation may eventually bring the attorney more business. The future possibility of press and acknowledgment does not forbid him to make such a solicitation. - Targeted direct mailed solicitation o Absent actual knowledge that the prospective client does not wish to receive communications from the lawyer, a lawyer is not prohibited from sending truthful, non-deceptive letters to persons known to face a specific legal problem o The outside of the envelope needs to say advertising material, and recorded and electronic communications must begin and end with the same thing - Group and prepaid legal service plans o A lawyer is permitted to participate in a group or prepaid legal service plan, even though the plan uses personal contacts and live telephone contacts to offer the plan to persons who are not known to need specific legal service. Rule 7.4 Communication of Fields of Practice and Specialization - A lawyer may state that they are a specialist as long as the certifying body has been approved by the state or the ABA. A lawyer may state that he does or does not practice in particular fields of law, but he may not say he is a specialist unless he is actually certified and that the certification is recognized Rule 7.5 Firm Names And Letterheads - A private law partnership may be designated by the names of one or more of the partners when partners die or retire, their names may be carried over to successor partnerships. I.E.: a law partnership may properly continue to practice under the name the Turner Partnership even though Turner may be retired. The firms letterhead may list turners name as a retired partner - Trade Names o Trade Names even ones that do not include the names of one or more partners are permitted, provided the name is not misleading and does not imply connection with a governmental agency or with a public or charitable legal services organization - Multistate Firms o A law firm that has offices in more than one jurisdiction may use the same name, internet address, or other professional designation in each jurisdiction However, when the lawyer in a particular office are identified, the identification must indicate the jurisdictional limitations on those lawyers not licensed in the jurisdiction where the office is located.  I.E.: If a firm has offices in NY, KY and TN the letterhead used in KY may list all of the lawyers in the firm, but it must indicate which partners are not licensed in KY. 24

False Indications of Partnership o Lawyers must not imply that they are partners or are otherwise associated with each other in a firm unless they really are Associated and Affiliated Law Firms o Two law firms may hold themselves out in the public as being associated or affiliated if they have a close, regular, ongoing relationship and if the designation is not misleading. The drawback is that the two firms may be treated as a single unit for conflict of interest purposes The name of lawyer holding a public office shall not be used in the name of a law firm, or in communication on its behalf, during any substantial period in which the lawyer is not actively and regularly practicing with the firm MAINTAINING THE INTEGRITY OF THE PROFESSION

Model Rule 8.1 Bar Admission & Disciplinary Matters -- Honesty is Key An applicant for admission to the bar must respond truthfully and completely t inquiries made on the application or otherwise by the admissions committee o You are required to correct bar authorities mistaken impressions regarding facts and to answer legal requests for data from bar authorities - False Statements o An applicant for admission to the bar, or a lawyer in connection with a bar admission application, must not knowingly make a false statement of material fact - Failure to disclose information o An applicant or a lawyer in connection with an applicants application for admissions to the bar, must not fail to disclose a fact necessary to correct a misapprehension known by the person to have arisen in the matter, or knowingly fail to respond to a lawful demand for information from an admissions authority. This rule does not require disclosure of information otherwise protected by the confidentiality provisions of the rules of professional conduct NOTES - Personal Conduct Prior to Application o Clear up all delinquent debts by paying in full or entering into and complying with a payment plan o If you have any character and fitness issues such as criminal convictions, bad debts, or bankruptcy, seek the advice of a lawyer o Bar cannot deny admissions because of views or beliefs

Rule 8.3 Reporting Professional Misconduct a. A lawyer who knows that another lawyer has committed a violation of the Rules of Professional Conduct that raises a substantial question as to that lawyer's honesty, trustworthiness or fitness as a lawyer in other respects, shall inform the appropriate professional authority. b. A lawyer who knows that a judge has committed a violation of applicable rules of judicial conduct that raises a substantial question as to the judge's fitness for office shall inform the appropriate authority. c. This Rule does not require disclosure of information otherwise protected by Rule 1.6 or information gained by a lawyer or judge while participating in an approved lawyers assistance program. NOTES - The legal profession prides itself on being self-policing. One element of a self-policing group is that each member of the group must be obligated to report misconduct by the other members. Therefore a lawyer who knows that another lawyer has violated the rules of professional conduct in such a way that raises substantial question as to the lawyers honesty, trustworthiness, or fitness as a lawyer must report the violation to the appropriate professional authority - Knowledge means actual knowledge, but it may be inferred from the circumstances. It has been held to more than mere suspicion. Thus, while a lawyer may report suspected misconduct, she must report known misconduct - Sanctions for Failure to Report Misconduct o A lawyer who fails to report this type of misconduct is herself subject to discipline for violating the rule requiring disclosure - Exception for Confidential Knowledge o This rule does not require disclosure of information protected by the confidentiality rules 1.6. Thus, fi a lawyer learns about another lawyers misconduct a privileged communication with the other lawyer or one of his clients, the lawyer has no duty to report the misconduct. The lawyer would be subject to discipline for violating the confidentiality rules if he did report it. 25

There is no duty to disclose information gained by a lawyer or judge while serving as a member of an approved lawyers assistance program that helps lawyers and judges with substance abuse programs

Rule 8.4 Misconduct It is professional misconduct for a lawyer to: - (a) You cannot violate or try to violate the MR o You cannot knowingly help or induce someone else to do so o You cannot do through someone else what you cannot do yourself under rules - (c) engage in conduct involving dishonesty, fraud, deceit or misrepresentation; NOTES - Certain Criminal Acts o A lawyer is subject to discipline for committing a criminal act that reflects adversely on his honest, trustworthiness, or fitness as a lawyer in other respects. To constitute professional misconduct, the crime must involve some characteristics that is relevant to the practice of law  EX: crimes involving dishonesty, breach of trust, substantial inference with the administration of justice, and most crimes involving violence, reflect on the lawyers fitness to practice law. Other crimes such as solicitation of prostitution, single offense of drunk driving, possession of marijuana, while punishable by law, does not necessarily trigger professional discipline o Dishonesty, Fraud, Deceit, or Misrepresentation  Any conduct involving the above listed constitutes professional misconduct y EX of misconduct that may not rise to the level of a crime, including cheating on a bar examination, defrauding own law firm by misusing expense accounts, plagiarism o Conduct prejudicial to the administration of justice  This rule is invoked when a lawyer, during representation of a client, knowingly, manifests bias based on race, sex, religion, national origin, disability, age, sexual orientation, socioeconomic status, and such action is prejudicial to the administrations of justice o Stating or implying ability to improperly influence officials  A lawyer must never state or imply that he has the ability to improperly influence a governmental agency or official or to achieve results by means that violate the law, or legal ethics rules o Assisting a judge in violation of judicial needs  A lawyer is subject to discipline for knowingly assisting a judge or judicial officer in conduct that violates the code of judicial conduct or other law Rule 8.5 Disciplinary Authority - A lawyer admitted to practice in this jurisdiction is subject to discipline o the authority if this jurisdiction, regardless of where the lawyers conduct occurs. If a lawyer provides or offers legal service in a jurisdiction where he is not admitted, he may be subject to the disciplinary authority of both his jurisdiction and the other jurisdiction for the same conduct. - Choice of law, in any exercise of the disciplinary authority of this jurisdiction, the rules of professional conduct to be applies shall be as follows: o For conduct in connection with a matter pending before a tribunal, the rules of the jurisdiction in which the tribunal sits, unless the riles of the tribunal provide otherwise o For any other conduct, the rules of the jurisdiction in which the lawyers conduct occurs, or, if the predominant effect of the conduct is in a different jurisdiction, the rules of that jurisdiction shall be applied to the conduct. A lawyer shall not be subject to discipline if the lawyers conduct conforms to the rules of a jurisdiction in which the lawyers reasonably believes the predominant effect of the lawyers conduct with occur.

26

S-ar putea să vă placă și